The Student Room Group

A-level Maths - ln(x) - Natural Logs... HELP?

Ok so last night I had trouble trying to figure out how this makes sense. I can't get my head round why this is...
Natural Logs.png

Can someone please explain? :frown:

Scroll to see replies

Original post by Philip-flop
Ok so last night I had trouble trying to figure out how this makes sense. I can't get my head round why this is...
Natural Logs.png

Can someone please explain? :frown:


You should've covered this in C2. It's one of the logarithm rules. Or are you asking for the proof?
(edited 7 years ago)
Reply 2
Original post by Philip-flop
Ok so last night I had trouble trying to figure out how this makes sense. I can't get my head round why this is...
Natural Logs.png

Can someone please explain? :frown:


It's the rule that alnx=lnxaa \ln x = \ln x^a.

Let y=lnxy = \ln x then x=eyx = e^y.

So xa=(ey)a=eayx^a = (e^y)^a = e^{ay}.

Take the logarithm of both sides lnxa=lneay=ay\ln x^a = \ln e^{ay} = ay.

So lnxa=alnx\ln x^a = a\ln x
Original post by Philip-flop
Ok so last night I had trouble trying to figure out how this makes sense. I can't get my head round why this is...
Natural Logs.png

Can someone please explain? :frown:


You know
ln(x) + ln(y) =ln(xy)
let y=x
ln(x) +ln (x) = ln(x^2)
2ln(x) = ln(x^2)
You can extend the rule (forwards and backwards) to end up with any rational power and it still works (for irrationals you need Zacken's method)
(edited 7 years ago)
Original post by RDKGames
You should've covered this in C2. It's one of the logarithm rules. Or are you asking for the proof?

Oh my god. Yeah of course!! I completely forgot! My Maths brain still hasn't switched on since the summer!

Original post by Zacken
It's the rule that alnx=lnxaa \ln x = \ln x^a.

Let y=lnxy = \ln x then x=eyx = e^y.

So xa=(ey)a=eayx^a = (e^y)^a = e^{ay}.

Take the logarithm of both sides lnxa=lneay=ay\ln x^a = \ln e^{ay} = ay.

So lnxa=alnx\ln x^a = a\ln x

Thank you Zacken!!
Can always rely on you for amazing explanations :smile:

Original post by ValerieKR
You know
ln(x) + ln(y) =ln(xy)
let y=x
ln(x) +ln (x) = ln(x^2)
2ln(x) = ln(x^2)
You can extend the rule to any power and it still works

Thank you so much!! Can't believe I forgot the "Powers of Logs" rule :colondollar:
Reply 5
Hi mate - I'll explain this stuff to you next week if you like ;-)
Original post by Paulxo
Hi mate - I'll explain this stuff to you next week if you like ;-)


Don't you worry, I have a looooooong list of questions I need to ask you :P
Reply 7
Original post by Philip-flop
Oh my god. Yeah of course!! I completely forgot! My Maths brain still hasn't switched on since the summer!


Thank you Zacken!!
Can always rely on you for amazing explanations :smile:


Thank you so much!! Can't believe I forgot the "Powers of Logs" rule :colondollar:


No worries
Ok I seem to be slipping up on Natural Logs again FFS. Why can't I do part a and c? :frown:

Solve to ln2.png

For part c I end up with x=ln(0.5)12x=\frac{\ln(0.5)-1}{2} :frown:
(edited 7 years ago)
Original post by Philip-flop
Ok I seem to be slipping up on Natural Logs again FFS. Why can't I do part a and c? :frown:

Solve to ln2.png

For part c I end up with x=ln(0.5)12x=\frac{\ln(0.5)-1}{2} :frown:


8=238=2^3

0.5=12=210.5=\frac{1}{2}=2^{-1}

Wait, how can you not do A and C when you've done B then...? I'm assuming you've done B since you're not asking for help on it...
(edited 7 years ago)
Original post by Philip-flop

For part c I end up with x=ln(0.5)12x=\frac{\ln(0.5)-1}{2} :frown:


^.-
Original post by RDKGames
8=238=2^3

0.5=12=210.5=\frac{1}{2}=2^{-1}

Wait, how can you not do A and C when you've done B then...? I'm assuming you've done B since you're not asking for help on it...


I think I worked out part b as a fluke :frown:
Here's my answer for part b...
Photo 02-09-2016, 22 21 01.jpg

Am I rubbish at Logs? :frown: :frown: :frown:
Original post by Philip-flop
I think I worked out part b as a fluke :frown:
Here's my answer for part b...


Am I rubbish at Logs? :frown: :frown: :frown:


It's technically correct but doesn't really flow well towards the end without converting the 4 into a power of 2.
Original post by Philip-flop
Ok I seem to be slipping up on Natural Logs again FFS. Why can't I do part a and c? :frown:

Solve to ln2.png

For part c I end up with x=ln(0.5)12x=\frac{\ln(0.5)-1}{2} :frown:


You're correct on part c - Remember that aln(x)= ln(x^a) even when a is negative

start part a by taking logs of both sides
Reply 14
On b, how did you turn ln 4/2 into ln 2?
Original post by Paulxo
On b, how did you turn ln 4/2 into ln 2?


Because 12×4\frac{1}{2}\times4? I don't know :frown:
Original post by ValerieKR
You're correct on part c - Remember that aln(x)= ln(x^a) even when a is negative

start part a by taking logs of both sides


Ok I've gone wrong with part a as well :frown:

Unparseable latex formula:

\loge^3x=\log8


Unparseable latex formula:

3x\loge=log8


Unparseable latex formula:

x=\frac{\log8}{3\loge}

Great. I don't know how to even write e^3x in latex. Here is my answer below...
Photo 02-09-2016, 23 27 28.jpg
I am aware that I am completely wrong with my answer so try to look past my stupidity and help me if you can :frown:
(edited 7 years ago)
Reply 18
Original post by Philip-flop
Great. I don't know how to even write e^3x in latex. Here is my answer below...
Photo 02-09-2016, 23 27 28.jpg
I am aware that I am completely wrong with my answer so try to look past my stupidity and help me if you can :frown:



This is correct. However, if this is Exercise 3B, Question 2A you need to give your answer in terms of ln2 \ln 2 .

Don't call yourself stupid just because you can't solve a math problem. Everyone who is assisting you in this thread has been doing maths way longer than you. So, they'll obviously be a little bit sharper at it...
Original post by Naruke
This is correct. However, if this is Exercise 3B, Question 2A you need to give your answer in terms of ln2 \ln 2 .

Don't call yourself stupid just because you can't solve a math problem. Everyone who is assisting you in this thread has been doing maths way longer than you. So, they'll obviously be a little bit sharper at it...


Oh yay. At least I kind of got the question right. But how do I give my answer in terms of ln2\ln 2 ?? :frown:

Quick Reply

Latest